Let a0<a1<a2… a_0 < a_1 < a_2 \ldots a0<a1<a2… be an infinite sequence of positive integers. Prove that there exists a unique integer n≥1 n \geq 1 n≥1 such that
an<a0+a1+…+ann≤an+1. a_n < \frac{a_0 + a_1 + \ldots + a_n } { n} \leq a_{n+1} .an<na0+a1+…+an≤an+1.
Note by Calvin Lin 6 years, 11 months ago
Easy Math Editor
This discussion board is a place to discuss our Daily Challenges and the math and science related to those challenges. Explanations are more than just a solution — they should explain the steps and thinking strategies that you used to obtain the solution. Comments should further the discussion of math and science.
When posting on Brilliant:
*italics*
_italics_
**bold**
__bold__
- bulleted- list
1. numbered2. list
paragraph 1paragraph 2
paragraph 1
paragraph 2
[example link](https://brilliant.org)
> This is a quote
This is a quote
# I indented these lines # 4 spaces, and now they show # up as a code block. print "hello world"
\(
\)
\[
\]
2 \times 3
2^{34}
a_{i-1}
\frac{2}{3}
\sqrt{2}
\sum_{i=1}^3
\sin \theta
\boxed{123}
Let's call Sk=a0+a1+...+ak S_k = a_0 + a_1 + ... + a_k Sk=a0+a1+...+ak and suppose Skk>ak+1 ∀m≤k \frac { { S }_{ k } }{ k } >{ a }_{ k+1 } \; \forall m \leq k kSk>ak+1∀m≤k Then we have k⋅Sk+Sk>k⋅ak+1+k⋅Sk⇔Skk>Sk+1k+1⇔Sk+1k+1<Skk<Sk−1k−1<...<S1=a0+a1 k\cdot { S }_{ k }+{ S }_{ k }>k\cdot { a }_{ k+1 }+k\cdot{ S }_{ k }\quad \Leftrightarrow \quad \frac { { S }_{ k } }{ k } >\frac { { S }_{ k+1 } }{ k+1 }\\ \quad \Leftrightarrow \quad \frac { { S }_{ k+1 } }{ k+1 } <\frac { { S }_{ k } }{ k } <\frac { { S }_{ k-1 } }{ k-1 } <...<{ S }_{ 1 }={ a }_{ 0 }+{ a }_{ 1 } k⋅Sk+Sk>k⋅ak+1+k⋅Sk⇔kSk>k+1Sk+1⇔k+1Sk+1<kSk<k−1Sk−1<...<S1=a0+a1 Hence, we prove by induction 2(a0+a1)>a0+a1+a2⇔a0+a1>a2kak+1<Sk<k(a0+a1)⇔ak+1<(a0+a1) 2(a_{ 0 }+{ a }_{ 1 })>{ a }_{ 0 }+{ a }_{ 1 }+{ a }_{ 2 }\quad \Leftrightarrow \quad { a }_{ 0 }+{ a }_{ 1 }>{ a }_{ 2 }\\ k{ a }_{ k+1 }<{ S }_{ k }<k(a_{ 0 }+{ a }_{ 1 })\quad \Leftrightarrow \quad { a }_{ k+1 }<(a_{ 0 }+{ a }_{ 1 }) 2(a0+a1)>a0+a1+a2⇔a0+a1>a2kak+1<Sk<k(a0+a1)⇔ak+1<(a0+a1) Because a0+a1 a_{ 0 }+{ a }_{ 1 } a0+a1 is a sum of integer number, there must be a unique number in the sequence for which aq+1≥a0+a1 a_{q+1} \geq a_{ 0 }+{ a }_{ 1 } aq+1≥a0+a1 and aq<a0+a1 a_{q} < a_{ 0 }+{ a }_{ 1 } aq<a0+a1. We now show that q q q is the required number. We have alredy proved that Sqq≤aq+1 \frac { { S }_{ q } }{q } \leq { a }_{ q+1 } qSq≤aq+1 by contraddiction! Thus we have only to show that aq<Sqq⇔q⋅aq−aq<Sq−aq⇔aq<Sq−1q−1{ a }_{ q }<\frac { S_{ q } }{ q } \quad \Leftrightarrow \quad q{ \cdot a }_{ q }-a_{ q }<S_{ q }-a_{ q }\quad \Leftrightarrow \quad { a }_{ q }<\frac { S_{ q-1 } }{ q-1 } aq<qSq⇔q⋅aq−aq<Sq−aq⇔aq<q−1Sq−1 And because Sq−1(q−1)+aq(q−1)<Sq−1+Sq−1(q−1)⇔Sqq<Sq−1q−1⇔Sqq<Sq−1q−1<Sq−2q−2<...<S1=a0+a1 S_{ q-1 }(q-1)+{ a }_{ q }(q-1)<S_{ q-1 }+S_{ q-1 }(q-1)\quad \Leftrightarrow \quad \frac { S_{ q } }{ q } <\frac { S_{ q-1 } }{ q-1 }\\ \quad \Leftrightarrow \quad \frac { S_{ q } }{ q } <\frac { S_{ q-1 } }{ q-1 } <\frac { S_{ q-2 } }{ q-2 } <...<S_{ 1 }={ a }_{ 0 }+{ a }_{ 1 } Sq−1(q−1)+aq(q−1)<Sq−1+Sq−1(q−1)⇔qSq<q−1Sq−1⇔qSq<q−1Sq−1<q−2Sq−2<...<S1=a0+a1 Excactly as done beofre we must have aq<a0+a1 a_q < a_0 + a_1 aq<a0+a1. That is true. And we are done.
Log in to reply
Beautiful solution
Question 1 was way easier than I expected it to be. Bummed that there were no proper number theory problems.
In recent years, they have made question 1 (and sometimes 4) pretty easy and direct to approach, so that all of the participants (who come from a wide range of ability level) have something to do, and have a chance of geting a HM.
This was indeed very nice, but also pretty easy, even for an IMO P1. I have posted my solution in AoPS.
http://www.artofproblemsolving.com/Forum/viewtopic.php?p=3542324&#p3542324
Here's a conjecture:
The last possible nnn that satisfies an<a0+a1+⋯+anna_n < \dfrac{a_0+a_1+\cdots +a_n}{n}an<na0+a1+⋯+an also satisfies a0+a1+⋯+ann≤an+1\dfrac{a_0+a_1+\cdots +a_n}{n}\le a_{n+1}na0+a1+⋯+an≤an+1. This is just a guess though.
These later posts will be what I have came up with.
Update: letting a0=xa_0=xa0=x and a1=ya_1=ya1=y, we find that in order for ak<a0+a1+⋯+akka_k < \dfrac{a_0+a_1+\cdots +a_k}{k}ak<ka0+a1+⋯+ak, we must have ak<x+ya_k < x+yak<x+y. However, since x+yx+yx+y is finite, and the sequence is strictly increasing, we must have that there is a maximum kkk that satisfies this condition.
So let kkk be the maximum such that it still satisfies the condition. Thus, ak+1≥x+ya_{k+1} \ge x+yak+1≥x+y. However, a0+a1+⋯+akk<x+y+(x+y)+⋯+(x+y)k=x+y\dfrac{a_0+a_1+\cdots +a_k}{k} < \dfrac{x+y+(x+y)+\cdots +(x+y)}{k}=x+yka0+a1+⋯+ak<kx+y+(x+y)+⋯+(x+y)=x+y. Thus, a0+a1+⋯+akk<ak+1\dfrac{a_0+a_1+\cdots +a_k}{k} < a_{k+1}ka0+a1+⋯+ak<ak+1.
This seems really sketchy though. Something seems wrong... one of my inequalities was strict when forming the right inequality, but the inequality in the problem is not strict...
EDIT: found out why mine was strict and the problem's wasn't. As long as the maximum kkk satisfies k≥2k\ge 2k≥2, then the upper bound inequality is strict, just like my result. However, the special case of the maximum kkk is k=1k=1k=1 yields a non-strict inequality.
TO DO: prove uniqueness of satisfying both inequalities.
I realize that I have to tighten the bound for each ak<x+ya_k < x+yak<x+y to something more strict, or something like that in order to prove the uniqueness of that one kkk that satisfies both inequalities. I feel like I'm almost there, but just can't get over the last little bump. I need to somehow use the ak<ak+1a_k < a_{k+1}ak<ak+1 to my advantage...
Problem Loading...
Note Loading...
Set Loading...
Easy Math Editor
This discussion board is a place to discuss our Daily Challenges and the math and science related to those challenges. Explanations are more than just a solution — they should explain the steps and thinking strategies that you used to obtain the solution. Comments should further the discussion of math and science.
When posting on Brilliant:
*italics*
or_italics_
**bold**
or__bold__
paragraph 1
paragraph 2
[example link](https://brilliant.org)
> This is a quote
\(
...\)
or\[
...\]
to ensure proper formatting.2 \times 3
2^{34}
a_{i-1}
\frac{2}{3}
\sqrt{2}
\sum_{i=1}^3
\sin \theta
\boxed{123}
Comments
Let's call Sk=a0+a1+...+ak and suppose kSk>ak+1∀m≤k Then we have k⋅Sk+Sk>k⋅ak+1+k⋅Sk⇔kSk>k+1Sk+1⇔k+1Sk+1<kSk<k−1Sk−1<...<S1=a0+a1 Hence, we prove by induction 2(a0+a1)>a0+a1+a2⇔a0+a1>a2kak+1<Sk<k(a0+a1)⇔ak+1<(a0+a1) Because a0+a1 is a sum of integer number, there must be a unique number in the sequence for which aq+1≥a0+a1 and aq<a0+a1. We now show that q is the required number. We have alredy proved that qSq≤aq+1 by contraddiction! Thus we have only to show that aq<qSq⇔q⋅aq−aq<Sq−aq⇔aq<q−1Sq−1 And because Sq−1(q−1)+aq(q−1)<Sq−1+Sq−1(q−1)⇔qSq<q−1Sq−1⇔qSq<q−1Sq−1<q−2Sq−2<...<S1=a0+a1 Excactly as done beofre we must have aq<a0+a1. That is true. And we are done.
Log in to reply
Beautiful solution
Question 1 was way easier than I expected it to be. Bummed that there were no proper number theory problems.
Log in to reply
In recent years, they have made question 1 (and sometimes 4) pretty easy and direct to approach, so that all of the participants (who come from a wide range of ability level) have something to do, and have a chance of geting a HM.
This was indeed very nice, but also pretty easy, even for an IMO P1. I have posted my solution in AoPS.
http://www.artofproblemsolving.com/Forum/viewtopic.php?p=3542324&#p3542324
Here's a conjecture:
The last possible n that satisfies an<na0+a1+⋯+an also satisfies na0+a1+⋯+an≤an+1. This is just a guess though.
These later posts will be what I have came up with.
Log in to reply
Update: letting a0=x and a1=y, we find that in order for ak<ka0+a1+⋯+ak, we must have ak<x+y. However, since x+y is finite, and the sequence is strictly increasing, we must have that there is a maximum k that satisfies this condition.
So let k be the maximum such that it still satisfies the condition. Thus, ak+1≥x+y. However, ka0+a1+⋯+ak<kx+y+(x+y)+⋯+(x+y)=x+y. Thus, ka0+a1+⋯+ak<ak+1.
This seems really sketchy though. Something seems wrong... one of my inequalities was strict when forming the right inequality, but the inequality in the problem is not strict...
EDIT: found out why mine was strict and the problem's wasn't. As long as the maximum k satisfies k≥2, then the upper bound inequality is strict, just like my result. However, the special case of the maximum k is k=1 yields a non-strict inequality.
TO DO: prove uniqueness of satisfying both inequalities.
Log in to reply
I realize that I have to tighten the bound for each ak<x+y to something more strict, or something like that in order to prove the uniqueness of that one k that satisfies both inequalities. I feel like I'm almost there, but just can't get over the last little bump. I need to somehow use the ak<ak+1 to my advantage...